Math, asked by chauhansanjeevkumar4, 7 months ago

jo karaga ma usha follow karuga aur jis sha nahi hoga bo mujha folllow karga​

Attachments:

Answers

Answered by Anonymous
11

\large\underline{\underline{Question:}}

\underline{\underline{Prove:}}

\dfrac{1 - tan^{2}\theta}{cot^{2}\theta} = tan^{2}\theta

______________________________________

\large\underline{\underline{To\:Find:}}

\text{To Prove that RHS = LHS}

______________________________________

\large\underline{\underline{We\:Know:}}

cot\:\theta = \dfrac{1}{tan\:\theta}

______________________________________

\color{red}\large\underline{\underline{Solution:}}

\dfrac{1 - tan^{2}\theta}{cot^{2}\theta} = tan^{2}\theta

\Rightarrow 1 - tan^{2}\theta = tan^{2}\theta \times (cot^{2}\theta - 1)

\Rightarrow 1 - tan^{2}\theta = tan^{2}\theta \times \bigg(\dfrac{1}{tan^{2}\theta} - 1\bigg)

\Rightarrow 1 - tan^{2}\theta = 1 - tan^{2}\theta

\therefore LHS = RHS

{\boxed{\therefore \dfrac{1 - tan^{2}\theta}{cot^{2}\theta} = tan^{2}\theta}}

Similar questions